subject
Business, 19.02.2021 01:00 jan1829

Which of the following statements is CORRECT? a. Suppose a firm's total assets turnover ratio falls from 1.0 to 0.9, but at the same time its profit margin rises from 9% to 10% and its debt increases from 40% of total assets to 60%. Under these conditions, the ROE will increase. b. Suppose a firm's total assets turnover ratio falls from 1.0 to 0.9, but at the same time its profit margin rises from 9% to 10%, and its debt increases from 40% of total assets to 60%. Under these conditions, the ROE will decrease. c. The modified DuPont equation provides information about how operations affect the ROE, but the equation does not include the effects of debt on the ROE. d. Other things held constant, an increase in the debt ratio will result in an increase in the profit margin on sales. e. Suppose a firm's total assets turnover ratio falls from 1.0 to 0.9, but at the same time its profit margin rises from 9% to 10% and its debt increases from 40% of total assets to 60%. Without additional information, we cannot tell what will happen to the ROE.

ansver
Answers: 2

Another question on Business

question
Business, 22.06.2019 03:30
Used cars usually have options: higher depreciation rate than new cars lower financing costs than new cars lower insurance premiums than new cars lower maintenance costs than new cars
Answers: 1
question
Business, 22.06.2019 13:10
The textbook defines ethics as “the principles of conduct governing an individual or a group,” and specifically as the standards one uses to decide what their conduct should be. to what extent do you believe that what happened at bp (british petrolium) is as much a breakdown in the company’s ethical systems as it is in its safety systems, and how would you defend your conclusion?
Answers: 2
question
Business, 22.06.2019 16:30
Which of the following has the largest impact on opportunity cost
Answers: 2
question
Business, 22.06.2019 20:00
Ryngard corp's sales last year were $38,000, and its total assets were $16,000. what was its total assets turnover ratio (tato)? a. 2.04b. 2.14c. 2.26d. 2.38e. 2.49
Answers: 1
You know the right answer?
Which of the following statements is CORRECT? a. Suppose a firm's total assets turnover ratio falls...
Questions
question
Mathematics, 19.04.2020 08:12
question
English, 19.04.2020 08:13
Questions on the website: 13722367